Đến nội dung

Hình ảnh

Topic về Bất đẳng thức, cực trị THCS


  • Please log in to reply
Chủ đề này có 1205 trả lời

#21
No Problem

No Problem

    Hạ sĩ

  • Thành viên
  • 67 Bài viết

Bài 6:(THCS) Cho $x,y,z>0$ và $x+y+z=1$. CMR: $x+2y+z\geq 4(x+y)(y+z)(z+x)$

Bài 7:(THCS) Với $a,b,c \in [0,1]$. Tìm GTLN của biểu thức: $A=\dfrac{a}{bc+1}+\dfrac{b}{ac+1}+\dfrac{c}{ab+1}$


Giả sử $0\le \ a\le \ b\le \ c\le \ 1$
:pe $bc+1\ge \ ac+1\ge \ ab+1$ và $(1-a)(1-b)\ge 0 $ :pe$ ab+1\ge \ a+b$
:Rightarrow $A\le \dfrac{a+b}{ab+1}+\dfrac{c}{ab+1}\le \ 1+c\le 2$
Đẳng thức xảy ra :D $(a;b;c)=(0;1;1)$ và các hoán vị
p/s:mình có ý kiến khi giải xong các bài thì nên nêu ra đk xảy ra dấu bằng :(

Bài viết đã được chỉnh sửa nội dung bởi No Problem: 03-07-2009 - 08:19


#22
No Problem

No Problem

    Hạ sĩ

  • Thành viên
  • 67 Bài viết

Bài 10:(THCS) Với $a,b,c$ là 3 cạnh của tam giác. CMR: $\dfrac{a}{\sqrt[3]{b^{3}+c^{3}}}+ \dfrac{b}{\sqrt[3]{a^{3}+c^{3}}}+ \dfrac{c}{\sqrt[3]{a^{3}+b^{3}}}<2\sqrt[3]{5}$

Các bài của THCS được ghi bên cạnh là chữ THCS, bài toán của THPT được ghi là THPT .
Rất mong các bạn post bài toán lên topic để mọi người cùng giải. Những công thức mình post có thể được dùng trực tiếp , nhưng khi đi thi thì ko thể được đâu mà phải chứng mình nó, trừ 2 BĐT là cô si và Bunhiacopxki.


$\dfrac{a}{\sqrt[3]{b^{3}+c^{3}}}\le \dfrac{a}{\sqrt[3]{\dfrac{(b+c)^3}{4}}}=\dfrac{\sqrt[3]{4}a}{b+c}$
mà $\sum\dfrac{a}{b+c}\le 2$
:Rightarrow $VT\le 2\sqrt[3]{4}$
p/s:LHS là cái gì vậy :( :D :pe :Rightarrow :pe :leluoi:

Bài viết đã được chỉnh sửa nội dung bởi No Problem: 03-07-2009 - 08:29


#23
No Problem

No Problem

    Hạ sĩ

  • Thành viên
  • 67 Bài viết
Bài 12[b] Cho$a,b,c>0$.ch/m:
$\dfrac{a^2(b+c)}{b^2+c^2}+\dfrac{b^2(c+a)}{c^2+a^2}+\dfrac{c^2(a+b)}{a^2+b^2}\ge a+b+c$
[b]Bài 13
Cho $a,b,c>0;ab+bc+ca=1$.Ch/m:
$\dfrac{1}{ab}+\dfrac{1}{bc}+\dfrac{1}{ca}\ge \ 3+\sqrt{1+\dfrac{1}{a^2}}+\sqrt{1+\dfrac{1}{b^2}}+\sqrt{1+\dfrac{1}{c^2}}$

#24
cvp

cvp

    Sĩ quan

  • Thành viên
  • 400 Bài viết

Bài 13 Cho $a,b,c>0;ab+bc+ca=1$.Ch/m:
$\dfrac{1}{ab}+\dfrac{1}{bc}+\dfrac{1}{ca}\ge \ 3+\sqrt{1+\dfrac{1}{a^2}}+\sqrt{1+\dfrac{1}{b^2}}+\sqrt{1+\dfrac{1}{c^2}}$

Dùng giả thiết $ab+bc+ca=1$
BĐT $<=>3+\dfrac{c(a+b)}{ab}+\dfrac{b(c+a)}{ca}+\dfrac{a(b+c)}{bc} \ge 3+\dfrac{\sqrt{(a+b)(a+c)}}{a}+\dfrac{\sqrt{(b+a)(b+c)}}{b}+\dfrac{\sqrt{(c+a)(c+b)}}{c}$
Chứng minh $\dfrac{c(a+b)}{ab}+\dfrac{b(c+a)}{ca}+\dfrac{a(b+c)}{bc}\ge \dfrac{\sqrt{(a+b)(a+c)}}{a}+\dfrac{\sqrt{(b+a)(b+c)}}{b}+\dfrac{\sqrt{(c+a)(c+b)}}{c}$
Đến đây dùng AM-GM: $\dfrac{c(a+b)}{ab}+\dfrac{b(c+a)}{ca}\ge 2\dfrac{\sqrt{(a+b)(a+c)}}{a}$
Tương tự => đpcm
Dấu = khi $a=b=c=\dfrac{1}{\sqrt{3}}$

Bài viết đã được chỉnh sửa nội dung bởi cvp: 03-07-2009 - 09:22

Hình đã gửi


#25
cvp

cvp

    Sĩ quan

  • Thành viên
  • 400 Bài viết
Mình đóng góp bài nè:
Bài 14: Cho a,b,c dương thỏa mãn abc=1. Chứng minh rằng:
$\dfrac{1}{b(5a+b)}+\dfrac{1}{c(5b+c)}+\dfrac{1}{a(5c+a)}\ge \dfrac{1}{2}$

Bài viết đã được chỉnh sửa nội dung bởi cvp: 03-07-2009 - 10:15

Hình đã gửi


#26
No Problem

No Problem

    Hạ sĩ

  • Thành viên
  • 67 Bài viết

Mình đóng góp bài nè:
Bài 14: Cho a,b,c dương thỏa mãn abc=1. Chứng minh rằng:
$\dfrac{1}{b(5a+b)}+\dfrac{1}{c(5b+c)}+\dfrac{1}{a(5c+a)}\ge \dfrac{1}{2}$


Đặt $a=\dfrac{x}{y},b=\dfrac{y}{z},c=\dfrac{z}{x}$
$Vt=\dfrac{x^2}{z^2+5xy}+\dfrac{y^2}{x^2+5yz}+\dfrac{z^2}{y^2+5xz}\ge \dfrac{(x+y+z)^2}{(x+y+z)^2+3(xy+yz+zx)}\ge \dfrac{1}{2}$
Đẳng thức xảy ra :neq a=b=c=1 :)

#27
cvp

cvp

    Sĩ quan

  • Thành viên
  • 400 Bài viết
Tiếp một bài dành cho THCS
Bài 15Cho $x,y,z$ dương.Chứng minh rằng :
$\dfrac{1}{x^2+yz}+\dfrac{1}{y^2+zx}+\dfrac{1}{z^2+xy}\le \dfrac{1}{2}(\dfrac{1}{xy}+\dfrac{1}{yz}+\dfrac{1}{zx}$

Hình đã gửi


#28
Hero Math

Hero Math

    Anh hùng của diễn đàn .

  • Thành viên
  • 237 Bài viết

Tiếp một bài dành cho THCS
Bài 15Cho $x,y,z$ dương.Chứng minh rằng :
$\dfrac{1}{x^2+yz}+\dfrac{1}{y^2+zx}+\dfrac{1}{z^2+xy}\le \dfrac{1}{2}(\dfrac{1}{xy}+\dfrac{1}{yz}+\dfrac{1}{zx}$

Cảm ơn anh Dũng và Minh đã đóng góp các bài tập.Áp dụng BĐT Cô si và BĐT: $ab+bc+ac\leq a^{2}+b^{2}+c^{2}. $ta có:
$VT=\dfrac{1}{x^2+yz}+\dfrac{1}{y^2+zx}+\dfrac{1}{z^2+xy}\leq \dfrac{1}{2}(\dfrac{1}{x\sqrt{yz}}+\dfrac{1}{y\sqrt{xz}}+\dfrac{1}{z\sqrt{xy}})$
$=\dfrac{1}{2}.\dfrac{\sqrt{xy}+\sqrt{yz}+\sqrt{xz}}{xyz}\leq \dfrac{1}{2}.\dfrac{x+y+z}{xyz}=\dfrac{1}{2}(\dfrac{1}{xy}+\dfrac{1}{yz}+\dfrac{1}{xz})=VP $

Bài viết đã được chỉnh sửa nội dung bởi Hero Math: 03-07-2009 - 20:06


#29
cvp

cvp

    Sĩ quan

  • Thành viên
  • 400 Bài viết
Hãy tiếp tục nào:
Bài 16:Cho $a,b,c$ dương thỏa mãn $a^2+b^2+c^2=1$Chứng minh rằng:
$\dfrac{1}{1-bc}+\dfrac{1}{1-ca}+\dfrac{1}{1-ab}\le \dfrac{9}{2}$
Bài 17:Cho $a,b,c$ dương thỏa mãn $a^2+b^2+c^2=1$Chứng minh rằng:
$a^2\sqrt{1-bc}+b^2\sqrt{1-ca}+c^2\sqrt{1-ab}\ge \sqrt{\dfrac{2}{3}}$

p/s: Hai bài toán có liên hệ đó!

Hình đã gửi


#30
- Nguyên Lê -

- Nguyên Lê -

    Hạ sĩ

  • Thành viên
  • 54 Bài viết

Bài 16:Cho $a,b,c$ dương thỏa mãn $a^2+b^2+c^2=1$Chứng minh rằng:
$\dfrac{1}{1-bc}+\dfrac{1}{1-ca}+\dfrac{1}{1-ab}\le \dfrac{9}{2}$
Bài 17:Cho $a,b,c$ dương thỏa mãn $a^2+b^2+c^2=1$Chứng minh rằng:
$a^2\sqrt{1-bc}+b^2\sqrt{1-ca}+c^2\sqrt{1-ab}\ge \sqrt{\dfrac{2}{3}}$

$\Leftrightarrow\dfrac{bc}{1-bc}+\dfrac{ca}{1-ca}+\dfrac{ab}{1-ab}\le\dfrac32$
Bài 16: Áp dụng bất đẳng thức AM - GM và Schwarz:
$\dfrac{bc}{1-bc}\le\dfrac{(b+c)^2}{4-2(b^2+c^2)}=\dfrac12.\dfrac{(b+c)^2}{2a^2+b^2+c^2}\le\dfrac12\left(\dfrac{b^2}{a^2+b^2}+\dfrac{c^2}{a^2+c^2}\right)$

Tương tự cộng lại được đpcm

Bài 17: Tương tự:
$a^2\sqrt{1-bc}=\dfrac{a^2}{\sqrt2}.\sqrt{2-2bc}\ge\dfrac{a^2}{\sqrt2}\sqrt{1+a^2+b^2+c^2-b^2-c^2}=\dfrac{a^2}{\sqrt2}.\sqrt{1+a^2}$
Áp dụng CBS ta có:
$\\\left(\dfrac13+1\right)(a^2+1)\ge\left(\dfrac a{\sqrt3}+1\right)^2\\\Rightarrow\sqrt{a^2+1}\ge\dfrac{a+\sqrt3}2\\\Rightarrow\dfrac{a^2}{\sqrt2}.\sqrt{1+a^2}\ge\dfrac{a^2(a+\sqrt3)}{2\sqrt2}$

Tương tự cộng lại, kết hợp với $a^3+b^3+c^3\ge\dfrac1{\sqrt3}$ nữa là được. =.=
(Hình như cách hơi lằng nhằng)

Bài viết đã được chỉnh sửa nội dung bởi - Nguyên Lê -: 04-07-2009 - 23:50


#31
AvidAbel_9x09

AvidAbel_9x09

    Hạ sĩ

  • Thành viên
  • 50 Bài viết
Hôm trc em thấy ở Quyên ôn thi đại học của ông anh có bài này cũng hay phết!:
"cho ba số ko âm a,b,c thỏa mãn: $ a+b+c=1$. Tìm GTLN của biểu thức: $ A= a^{2}b+b^{2}c+c^{2}a $"
Em nghĩ đây là một bài hay nên mời các tiền bối cứ làm càng nhìu cách càng tốt giúp em! :)

#32
cvp

cvp

    Sĩ quan

  • Thành viên
  • 400 Bài viết
Lời giải của bạn ổn rùi.Mình xin đóng góp cách khác cho bài 2 để thấy sự liên hệ:
Áp dụng bđt Trê bư sép:
$a^2\sqrt{1-bc}+b^2\sqrt{1-ca}+c^2\sqrt{1-ab}\ge \dfrac{1}{3}(a^2+b^2+c^2)(\sqrt{1-bc}+\sqrt{1-ca}+\sqrt{1-ab})$
Vì vậy cần cm: $\sqrt{1-bc}+\sqrt{1-ca}+\sqrt{1-ab}\ge \sqrt 6$
Đến đây áp dụng CBS ta có $(\sqrt{1-bc}+\sqrt{1-ca}+\sqrt{1-ab})^2(\dfrac{1}{1-bc}+\dfrac{1}{1-ca}+\dfrac{1}{1-ab}\ge 27$
Sử dụng kết quả bài trên =>$\sqrt{1-bc}+\sqrt{1-ca}+\sqrt{1-ab}\ge \sqrt 6$
=>đpcm!
Dấu = khi $a=b=c=\dfrac{1}{\sqrt{3}}$

Hình đã gửi


#33
admireM

admireM

    Binh nhất

  • Thành viên
  • 49 Bài viết

Hôm trc em thấy ở Quyên ôn thi đại học của ông anh có bài này cũng hay phết!:
"cho ba số ko âm a,b,c thỏa mãn: $ a+b+c=1$. Tìm GTLN của biểu thức: $ A= a^{2}b+b^{2}c+c^{2}a $"
Em nghĩ đây là một bài hay nên mời các tiền bối cứ làm càng nhìu cách càng tốt giúp em! :wub:


Bài này khó làm sao mà lại thi đại học sao được, Bạn cứ nói là bài này bạn nghĩ ra cho nhanh:)

Giả sử a max, có các trường hợp sau:

a :wub: b :beat c

Khi đó $ a^2b+b^2c+ c^2a $ :ech $ (a+c)^2b $ :ech $4/27 [(a+c)/2 + (a+c)/2 +b]^3 $
=4/27

Trường hợp a :beat c :ech b khó chưa nghĩ ra đợi đấy.

Bài viết đã được chỉnh sửa nội dung bởi admireM: 05-07-2009 - 11:41


#34
admireM

admireM

    Binh nhất

  • Thành viên
  • 49 Bài viết

Bài này khó làm sao mà lại thi đại học sao được, Bạn cứ nói là bài này bạn nghĩ ra cho nhanh:)

Giả sử a max, có các trường hợp sau:

a :beat b :beat c

Khi đó A= $ a^2b+b^2c+ c^2a $ :ech $ (a+c)^2b $ :leq $4/27 [(a+c)/2 + (a+c)/2 +b]^3 $
=4/27

Trường hợp a :ech c :ech b khó chưa nghĩ ra đợi đấy.


Với trường hợp này:
A :leq $ (a+c/2)^2(b+c/2) $ :wub: $ 4/27[(a+c/2)/2+(a+c/2)/2 + (b+c/2)]^3 $ =4/27

Vậy A :wub: 4/27
Dấu bằng xảy ra khi a=2/3, b=1/3, c=0

Vậy Amax=4/27

Bài viết đã được chỉnh sửa nội dung bởi admireM: 05-07-2009 - 12:26


#35
admireM

admireM

    Binh nhất

  • Thành viên
  • 49 Bài viết
Ủng hộ một bài BDT hay (nhưng hơi khó):

Cho a, b, c là các số không âm thõa mãn: a+b+c=3
Chứng minh rằng $a^2+b^2+c^2+\dfrac{3}{2}abc$ :wub: $\dfrac{9}{2}$

Bài viết đã được chỉnh sửa nội dung bởi Nguyễn Hoàng Nam: 06-07-2009 - 17:52


#36
dungbo_213

dungbo_213

    Binh nhì

  • Thành viên
  • 14 Bài viết

Ủng hộ một bài BDT hay (nhưng hơi khó):

Cho a, b, c là các số không âm thõa mãn: a+b+c=3
Chứng minh rằng $a^2+b^2+c^2+(3/2)abc$ :wub: 9/2


biến đổi $p,q,r$

$2p^2-4q+3r \ge 9$
$9-4q+3r \ge 0$ (đúng với Schur )
Đường còn dài… và còn nhiều hơn chông gai…

Rồi thì lặng lẽ những tháng ngày buồn ở lại… ngày vui dễ lắng… mau phai…

Mai về nhìn lại cuộc đời vui ghê… về nhìn lại yêu thương vẫn thế… giữa cơn đau nặng nề… khốn

khó lê thê…

#37
cvp

cvp

    Sĩ quan

  • Thành viên
  • 400 Bài viết
Tiếp tục bài này:
Cho $a,b,c$ là ba cạnh của một tam giác thỏa mãn: $a^2+b^2+c^2=3$
Chứng minh rằng: $a+b+c\ge 2+abc$

Hình đã gửi


#38
Nguyễn Hoàng Nam

Nguyễn Hoàng Nam

    Độc thân...

  • Thành viên
  • 334 Bài viết
Chứng mình rằng:
$\dfrac{2}{b+c}+\dfrac{2}{a+b}+\dfrac{2}{a+c} \geq \dfrac{9}{a+b+c}$

Bài viết đã được chỉnh sửa nội dung bởi Nguyễn Hoàng Nam: 08-07-2009 - 09:55

Kho tư liệu bất đẳng thức

My blog

My website
Bán acc Megaupload giá rẻ, giảm giá đặc biệt cho các thành viên của VMF :D
Contact: 01644 036630

#39
dungbo_213

dungbo_213

    Binh nhì

  • Thành viên
  • 14 Bài viết

Chứng mình rằng:
$\dfrac{2}{b+c}+\dfrac{2}{a+b}+\dfrac{2}{a+c} \geq \dfrac{2}{a+b+c}$


đề sai ,vế phải là 9
sử dụng Svac
Đường còn dài… và còn nhiều hơn chông gai…

Rồi thì lặng lẽ những tháng ngày buồn ở lại… ngày vui dễ lắng… mau phai…

Mai về nhìn lại cuộc đời vui ghê… về nhìn lại yêu thương vẫn thế… giữa cơn đau nặng nề… khốn

khó lê thê…

#40
congcomMật khẩu:

congcomMật khẩu:

    Hạ sĩ

  • Thành viên
  • 84 Bài viết

Dùng giả thiết $ab+bc+ca=1$
BĐT $<=>3+\dfrac{c(a+b)}{ab}+\dfrac{b(c+a)}{ca}+\dfrac{a(b+c)}{bc} \ge 3+\dfrac{\sqrt{(a+b)(a+c)}}{a}+\dfrac{\sqrt{(b+a)(b+c)}}{b}+\dfrac{\sqrt{(c+a)(c+b)}}{c}$
Chứng minh $\dfrac{c(a+b)}{ab}+\dfrac{b(c+a)}{ca}+\dfrac{a(b+c)}{bc}\ge \dfrac{\sqrt{(a+b)(a+c)}}{a}+\dfrac{\sqrt{(b+a)(b+c)}}{b}+\dfrac{\sqrt{(c+a)(c+b)}}{c}$
Đến đây dùng AM-GM: $\dfrac{c(a+b)}{ab}+\dfrac{b(c+a)}{ca}\ge 2\dfrac{\sqrt{(a+b)(a+c)}}{a}$
Tương tự => đpcm
Dấu = khi $a=b=c=\dfrac{1}{\sqrt{3}}$

bài này nếu em nhớ ko nhầm thì ở trong toán học tuổi trẻ đúng ko hở heromath
cuộc đời ko bao giờ giữ lòng tự trọng cho bạn mà ban phải tự tạo ra nó




13 người đang xem chủ đề

0 thành viên, 13 khách, 0 thành viên ẩn danh